5. Übung am 9. Mai 2008

Benutzeravatar
ibi
Dr. h.c.
Beiträge: 443
Registriert: 12.10.2006, 20:34
Wohnort: Kagran / Donaustadt

5. Übung am 9. Mai 2008

Beitrag von ibi »

Zur Erinnerung: Freitag nächste Woche (9. Mai) ist Test UND Tutorium.
Ist das wirklich deren Ernst?
David Seppi

Gott ist theoretischer Physiker

Benutzeravatar
pat
Beiträge: 418
Registriert: 07.10.2006, 16:11
Kontaktdaten:

Re: 5. Übung am 9. Mai 2008

Beitrag von pat »

Es sieht sehr danach aus. Aber morgen werden wir wohl Klarheit haben.
Ist aber vielleicht nicht einmal so schlecht, wenn man sich mit den Beispielen beschäftigt, dann werden sie durchgerechnet und kommen zum Test ;)
Wenn man sich das allerdings so ansieht, dürfte das vom Vormittag nicht am Nachmittag zum Test kommen - sonst hätten sie beim zweiten Test nur 3 Übungen als Stoffgebiet.

Um mal die Beispiele anzusprechen: Hab ich beim ersten Beispiel was falsch verstanden oder setze ich wirklich nur in Formel (3.95) für die Zustandssumme, (3.91) für die freie Energie, (3.48) für die Entropie und (3.99) für die innere Energie ein?
Das hätte irgendwie Physik-2-"Wir wollen, dass sie die Zwischenschritte im Demtröder durchrechnen"-Niveau...
Ich hab das Forum lieb, weil es schon so lange da ist und man auch Infos von höheren Semestern bekommt :)

Benutzeravatar
ibi
Dr. h.c.
Beiträge: 443
Registriert: 12.10.2006, 20:34
Wohnort: Kagran / Donaustadt

Re: 5. Übung am 9. Mai 2008

Beitrag von ibi »

Wundert's Dich?
Bis jetzt waren einige Statistik-Beispiele so.

Na gut, wir sehn uns morgen ...
David Seppi

Gott ist theoretischer Physiker

Teonanacatl
Beiträge: 53
Registriert: 08.11.2007, 19:06

Re: 5. Übung am 9. Mai 2008

Beitrag von Teonanacatl »

das tutorium am 9.5. ist nicht mehr stoff zum ersten test.

mfg

Benutzeravatar
ibi
Dr. h.c.
Beiträge: 443
Registriert: 12.10.2006, 20:34
Wohnort: Kagran / Donaustadt

Re: 5. Übung am 9. Mai 2008

Beitrag von ibi »

Gut, das 1. Bsp war ja ned schwer, aber hat jemand eine Idee fürs 2.?
2b) ist mir relativ klar. Da die Verteilung binomial ist, ist der Erwartungswert einfach N/2 up und N/2 down, damit dann M berechnen.
Aber wie ich auf den Hamiltonoperator komme, den ich für 2a brauch, ist mir nicht klar.
David Seppi

Gott ist theoretischer Physiker

Benutzeravatar
Khalidah
Lektorin
Beiträge: 145
Registriert: 09.10.2006, 14:25

Re: 5. Übung am 9. Mai 2008

Beitrag von Khalidah »

schau mal auf p69...
ich habs zumindest mit den formeln zur "quantum mechanical description" von kanonischen systemen gelöst...
Glücklich ist der, der nicht nur machen kann was er will, sondern auch wirklich will was er macht.

Benutzeravatar
ibi
Dr. h.c.
Beiträge: 443
Registriert: 12.10.2006, 20:34
Wohnort: Kagran / Donaustadt

Re: 5. Übung am 9. Mai 2008

Beitrag von ibi »

He, toll, da steht eh alles was ich brauch!
Danke!
David Seppi

Gott ist theoretischer Physiker

bodymurat
Beiträge: 2
Registriert: 07.05.2008, 17:36

Re: 5. Übung am 9. Mai 2008

Beitrag von bodymurat »

kann jemand bitte seine lösung online stellen?

Benutzeravatar
ibi
Dr. h.c.
Beiträge: 443
Registriert: 12.10.2006, 20:34
Wohnort: Kagran / Donaustadt

Re: 5. Übung am 9. Mai 2008

Beitrag von ibi »

Mein Ansatz zu 2a (kann das wer bestätigen?):

Gleichung 3.133 und Gleichung 3.134 ergeben kombiniert:

Z_c = \underset{n}{\sum}e^{-\beta E_n} (H ist ja diagonal)

Um an die Eigenenergie eines bestimmten Zustands zu kommen, rechne ich:

E_n = \left( \frac{n\hbar}{2} - \frac{(N-n)\hbar}{2} \right)B = \left( n\hbar - \frac{N\hbar}{2} \right)B wobei n Teilchen spin-up und N-n Teilchen spin-down haben (N ist die Gesamtzahl der Teilchen).

Da meine Energien aber entartet sind, muß ich in der obigen Summenformel jeden Summanden mit seiner Entartung gewichten.
Das Ganze ist eine Binomialverteilung, die Entartung ist also N über n.

Ich erhalte also: Z_c = \sum_{n=0}^{N} {N \choose n} e^{-\beta \left( n\hbar - \frac{N\hbar}{2} \right)B}

Kann das wer bestätigen?
Vor allem: Kann man das irgendwie vereinfachen? Wie?
David Seppi

Gott ist theoretischer Physiker

m0tzerl
Beiträge: 89
Registriert: 06.11.2006, 00:16

Re: 5. Übung am 9. Mai 2008

Beitrag von m0tzerl »

ibi hat geschrieben:Mein Ansatz zu 2a (kann das wer bestätigen?):

Gleichung 3.133 und Gleichung 3.134 ergeben kombiniert:

Z_c = \underset{n}{\sum}e^{-\beta E_n} (H ist ja diagonal)

Um an die Eigenenergie eines bestimmten Zustands zu kommen, rechne ich:

E_n = \left( \frac{n\hbar}{2} - \frac{(N-n)\hbar}{2} \right)B = \left( n\hbar - \frac{N\hbar}{2} \right)B wobei n Teilchen spin-up und N-n Teilchen spin-down haben (N ist die Gesamtzahl der Teilchen).

Da meine Energien aber entartet sind, muß ich in der obigen Summenformel jeden Summanden mit seiner Entartung gewichten.
Das Ganze ist eine Binomialverteilung, die Entartung ist also N über n.

Ich erhalte also: Z_c = \sum_{n=0}^{N} {N \choose n} e^{-\beta \left( n\hbar - \frac{N\hbar}{2} \right)B}

Kann das wer bestätigen?
Vor allem: Kann man das irgendwie vereinfachen? Wie?
mein ergebnis lautet: Z_k=cosh((B*\gamma*h)/(2*k_B*T))

ergibt sich aus: Z_k=Tr(e^{-\beta*H})
und: H=-\gamma*B*S_z

berechnung von Zk durch diese formeln ergibt das genannte ergebnis. 2b) hab ich gerade versucht, kommt zwar was lustiges raus, aber weit entfernt von irgendetwas mit curie beweisen...
Zuletzt geändert von m0tzerl am 07.05.2008, 20:15, insgesamt 1-mal geändert.

Benutzeravatar
ibi
Dr. h.c.
Beiträge: 443
Registriert: 12.10.2006, 20:34
Wohnort: Kagran / Donaustadt

Re: 5. Übung am 9. Mai 2008

Beitrag von ibi »

Dein Ergebnis ist aber verdächtig unabhängig von N.
David Seppi

Gott ist theoretischer Physiker

m0tzerl
Beiträge: 89
Registriert: 06.11.2006, 00:16

Re: 5. Übung am 9. Mai 2008

Beitrag von m0tzerl »

das ist richtig. kann auch kompletter blödsinn sein. aber wer sagt dass Zk von N abhängen muss / sollte ? dieses "nicht-wechselwirkend" könnte auch darauf hindeuten, dass es eben unabhängig sein muss... oder es is überhaupt ganz anders :)

Benutzeravatar
ibi
Dr. h.c.
Beiträge: 443
Registriert: 12.10.2006, 20:34
Wohnort: Kagran / Donaustadt

Re: 5. Übung am 9. Mai 2008

Beitrag von ibi »

Zumindest der Hamilton-Operator sollte doch von N abhängen.
David Seppi

Gott ist theoretischer Physiker

m0tzerl
Beiträge: 89
Registriert: 06.11.2006, 00:16

Re: 5. Übung am 9. Mai 2008

Beitrag von m0tzerl »

stimmt, das ändert mein ergebnis auf Z_k = 2*cosh((\gamma*B*h)/(2*k_b*T))

setzt man jetzt mit <M>=Tr(M*r_c)
ein, komme ich auf <M>=-(N*\gamma*h)/2*tanh((\gamma*B*h)/(2*k_b*T))

tja, da seh ich irgendwie den beweis des curie'schen gesetzes nicht. vielleicht irgendwer außer mir?
oder ich bin komplett auf dem holzweg, was natürlich auch sein kann.
dennoch denke ich, dass man das quantenmechanisch rechnen muss und man klassisch wohl wenig chancen hat.

Benutzeravatar
Khalidah
Lektorin
Beiträge: 145
Registriert: 09.10.2006, 14:25

Re: 5. Übung am 9. Mai 2008

Beitrag von Khalidah »

@m0tzerl: wenn ein physiker nicht mehr weiter weiß macht er taylorentwicklung... hat zumindest bei mir geholfen ;)
Glücklich ist der, der nicht nur machen kann was er will, sondern auch wirklich will was er macht.

Antworten

Zurück zu „Statistische Physik I“